Find a_10 5, 12, 19, 26, 33...

Answers

Answer 1

Given,

The progression is 5, 12, 19, 26, 33...​

The first term of the series is, a = 5.

The common difference of the series is,

d = 12 - 5 = 7

The 10th term of the series is,

[tex]\begin{gathered} a_{10}=a+(10-1)\times d \\ =5+9\times7 \\ =5+63 \\ =68 \end{gathered}[/tex]

Hence, the 10th term of the series is 68.


Related Questions

You deposit $300 each month into an account earning 6% interest compounded monthly. a) How much will you have in the account in 30 years? $ b) How much total money will you put into the account? $ c) How much total interest will you earn?

Answers

Using the future value formula, it is found that:

a) You will have $301,355 in the account after 30 years.

b) You will have put $108,000 in the account.

c) You will have earned $193,355 in interest.

What is the future value formula?

The future value formula  is given by:

[tex]V(n) = P\left[\frac{(1 + r)^n - 1}{r}\right][/tex]

In which the parameters are described as follows:

P is the payment.n is the number of payments.r is the interest rate.

Considering the $300 deposits, the interest rate of 6% and the monthly compoundings for 30 years, the values of the parameters are given as follows:

P = 300, r = 0.06/12 = 0.005, n = 12 x 30 = 360.

Hence the balance of the account after 30 years will be given by:

[tex]V(30) = 300\left[\frac{(1.005)^{360} - 1}{0.005}\right][/tex]

V(30) = $301,355.

The amount placed into the account is given by:

360 x 300 = $108,000.

The remaining amount is earned in interest, hence:

301355 - 108000 = $193,355.

More can be learned about the future value formula at https://brainly.com/question/24703884

#SPJ1

Question 12 of 47Solve kx - 2 = 7 for x.

Answers

ANSWER:

D. x = 9/k

STEP-BY-STEP EXPLANATION:

We have the following equation:

[tex]kx-2=7[/tex]

We solving for x:

[tex]\begin{gathered} kx=7+2 \\ \\ x=\frac{9}{k} \end{gathered}[/tex]

The correct answer is D. x = 9/k

Factor the expression over the complex numbers. x^2+25 Enter your answer in the box.
please help

Answers

The complex number factors are (x+5i) and (x-5i).

The term a + bi, where a and b are real numbers, can be used to represent any complex number.

A complex number is a component of a number system that includes an element with the symbol I, sometimes known as the imaginary unit, and that extends the real numbers by satisfying the equation i² = -1. Since no real number can fulfil the conditions in the previous equation, i was referred to as an imaginary number. The complex number a + bi is known as having real and imaginary parts, respectively, a and b. The group of complex numbers is represented by the letter C.

The given expression is x² + 25

Now this can be written as :

x² - [(i)²×(5)²] as we know that i² = -1.

Now we use the algebraic form of a²-b² =(a + b)(a - b) to separate it into complex factors.

or , (x+5i)(x-5i)

Therefore the complex number factors are (x+5i) and (x-5i).

To learn more about complex numbers visit:

https://brainly.com/question/10251853

#SPJ1

Find missing side length

Answers

Answer:

7

Step-by-step explanation:

42 divided by 3 = 14 = total of triangle XYZ

14 divided by 2 = 7

line X/Y = 7

Which Similarity Theorem is described by the text below?If the lengths of three sides of one triangle are proportional to the lengths of three sides ofanother, then the triangles are similar.A. AAB. ASAC. SASD. SSS

Answers

Answer:

SSS.

Step-by-step explanation:

The text above described the Side-Side-Side Theorem, which states that two triangles are congruent if three sides of one are equal or proportional respectively to three sides of the other (SSS=SSS).

The add the letters together and get the same format that u wood get if u whereby

A pair of dice is rolled 3 times. What is the probability that a sum of 9 on the 2 dice will occur at least once?

Answers

The probability that a sum of 9 on the 2 die will occur at least once is 217/729.

Sum of 9 in a pair of dice can occur in the following 4 ways.

3 + 6

4 + 5

5 + 4

6 + 3

Also, total number of ways = 6*6 = 36

Hence, probability of getting a sum of 9  = 4/36 = 1/9.

We know that,

Probability of getting a sum of 9 + Probability of not getting a sum of 9 = 1

Hence, we can write,

1/9 +  Probability of not getting a sum of 9 = 1

Probability of not getting a sum of 9 = 8/9

Also,

Probability of getting the sum of 9 atleast 1 + probability of getting the sum 9 none of the times = 1

Hence, we can write,

Probability of getting the sum of 9 atleast once =  1 - probability of getting the sum 9 none of the times/

Probability of getting the sum of 9 atleast 1 = 1 - (8/9)*(8/9)*(8/9) = 1 - 512/729 = 217/729.

To learn more about probability, here:-

https://brainly.com/question/11234923

#SPJ1

Select all equations that are equivalent to x² + 6x = 16

Answers

Answer

Options C and F are correct.

x² + 6x + 9 = 25

(x + 3)² = 25

are both equivalent to

x² + 6x = 16

Explanation

The equations that are equivalent to x² + 6x = 16 will all have to reduce to this exact equation on simplification.

x² + 6x + 9 = 0

Option A is not correct as this equation isn't equal to the equation given in the question.

x² + 6x + 9 = 16

x² + 6x = 16 - 9

x² + 6x = 7

Option B is not correct as this equation isn't equal to the equation given in the question.

x² + 6x + 9 = 25

x² + 6x = 25 - 9

x² + 6x = 16

This option C is corect as it matches the equation given in the quesion.

(x + 3)² = 0

x² + 6x + 9 = 0

Option D is not correct as this equation isn't equal to the equation given in the question.

(x + 3)² = 16

x² + 6x + 9 = 16

x² + 6x = 16 - 9

x² + 6x = 7

Option E is not correct as this equation isn't equal to the equation given in the question.

(x + 3)² = 25

x² + 6x + 9 = 25

x² + 6x = 25 - 9

x² + 6x = 16

This option F is corect as it matches the equation given in the quesion.

Hope this Helps!!!

Jason jumped off of a cliff into the ocean in Acapulco while vacationing with some friends. His height above ocean measured in feet as a function of time could be modeled by the function h(t) = -16t2 + 16t + 672, where t is the time in seconds and h is the height in feet. After how many seconds did Jason hit the water

Answers

You have the following function for the height respect to the water, Jason has after time t he jumped:

[tex]h(t)=-16t^2-16t+672[/tex]

In order to determine the time Jason takes to hit the water, equal h = 0. Thus, you have a quadratic equation and it is necessary to find the zeros of the polynomial:

[tex]0=-16t^2-16t+672[/tex]

use the following quadratic formula to find the zeros:

[tex]t=\frac{-b\pm\sqrt[]{b^2-4ac}}{2a}[/tex]

where a,b and c are the coefficients of the polynomial:

a = -16

b = -16

c = 672

replace these values into the formula:

[tex]\begin{gathered} t=\frac{-(-16)\pm\sqrt[]{(-16)^2-4(-16)(672)}}{2(-16)} \\ t=\frac{16\pm\sqrt[]{43264}}{-32} \\ t=\frac{16\pm208}{-32} \\ t=-0.5\pm(-6.5) \end{gathered}[/tex]

Then, you obtain the following two values:

[tex]\begin{gathered} t=-7 \\ t=6 \end{gathered}[/tex]

Select the positive values because negative time does not have physical meaning.

Hence, Jason takes 6 seconds to hit the water.

What is the equal number for 2 tens and 15 ones

Answers

Two tens are 20 plus 15 ones is 35!

Sean is hiking the Appalachian Trail from Georgia to Maine. The distance of his hike is 2200 miles. It
took Sean 134 days to complete the hike. The data below represent the distance, D, he had hiked t
days after the start of his trip.
t
D(0)
0
0
19
308
In
34
578
67
1070
1. Determine the linear regression equation for this data.
Linear Regression Equation: D =
Round to the nearest tenth as needed.
2. Interpret the meaning of the slope of this regression model.
Sean hiked at a rate of approximately
91
1715
miles.
112
1902
Select an answer
3. Use your linear regression equation to estimate the total number of miles Sean had hiked in 52
days. Round your answer to the nearest mile.
In 52 days, Sean had hiked
4. Use your linear regression equation to estimate the total number of days it took for Sean to hike
377 miles. Round your answer to the nearest day.
days, Sean had hiked 377 miles.
134
2200

Answers

From the given table, it is found that:

1. The linear regression equation is: y = 16.96x + 3.

2. The meaning of the slope is that he hiked an average of 16.96 miles a day.

3. In 52 days, Sean had hiked 885 miles.

4. In 22 days, Sean had hiked 377 miles.

How to find the equation of linear regression?

To find the regression equation, also called line of best fit or least squares regression equation, we need to insert the points (x,y) in the calculator. These points are given on a table or in a scatter plot in the problem.

In the context of this problem, the points are given as follows:

(0,0), (19,308), (34, 578), (67, 1070), (91, 1715), (112, 1902), (134, 2200).

Using a calculator, the equation is given as follows:

y = 16.96x + 3.

The input of the function and the output of the function are given as follows:

Input: number of days.Output: number of miles hiked.

The slope is the rate of change, that is, change in the output divided by change in the input. Hence the slope of 16.96 means that he hiked an average of 16.96 miles a day.

In 52 days, the number of miles hiked is:

y = 16.96(52) + 3 = 885 miles.

The number of days is took to hike 377 miles is found as follows:

377 = 16.96x + 3

x = 374/16.96

x = 22 days.

More can be learned about linear regression at https://brainly.com/question/22992800

#SPJ1

The answers are 0, 1, 3, and 3.5. I think the answer is 3 but I'm not sure

Answers

The y-intercept of any function is the point where the line intersect with the y-axis, at this point the value of x is zero.

If you look at the graph, the line intersects the y-axis at point (0,3), so the y-coordinate of the y-intercept is 3

question 11, rewrite the formulaThe area of a triangle is A-a. Rewrite the formula for b.b. Rewrite the formula for h.

Answers

Given the formula:

[tex]A\text{ = }\frac{1}{2}bh[/tex]

a) Rewrite for b:

First step: Multiply 2 to both sides

[tex]\begin{gathered} 2A\text{ = }\frac{1}{2}bh\text{ }\ast2 \\ \\ 2A\text{ = bh} \end{gathered}[/tex]

Second step: divide both sides by h

[tex]\frac{2A}{h}\text{ = b}[/tex][tex]\begin{gathered} \text{Thus,} \\ b\text{ = }\frac{2A}{h} \end{gathered}[/tex]

B) Rewrite for h:

First step: Multiply 2 to both sides

[tex]\begin{gathered} 2A\text{ = }\frac{1}{2}bh\ast2 \\ \\ 2A\text{ = bh} \end{gathered}[/tex]

Second step: divide both sides by b

[tex]\begin{gathered} \frac{2A}{b}\text{ = }\frac{bh}{b} \\ \\ \frac{2A}{b}\text{ = h} \end{gathered}[/tex][tex]h\text{ = }\frac{2A}{b}[/tex]

There are 15 members in a bicycle club this fall by next summer Bing home saying number of members will be 140% of this number what is the main goal for the number of members next summer 

Answers

The main goal is 140 percent i.e., 21 members.

What is meant by percentage?

In mathematics, a percentage (from Latin: percentage , "by a hundred") may be a number or ratio expressed as a fraction of 100. it's often denoted using the percent sign, "%", although the abbreviations "pct.", "pct" and sometimes "pc" also are used. A percentage may be a dimensionless number (pure number); it has no unit of measurement. The term "percent" springs from the Latin per centum, meaning "hundred" or "by the hundred". The Italian word per cento, which means "for a hundred," gradually shrank to become the symbol for "percent." The "per" was often abbreviated as "p."—eventually disappeared entirely. The "cento" was contracted to 2 circles separated by a horizontal line, from which the fashionable "%" symbol is derived.

140 percent of 15 = 15x140/100 = 21 members

Hence, the main goal is 21 members.

To know more about percentage, click on the link

https://brainly.com/question/1053881

#SPJ9

Select the correct answer.
What is the sum of this expression?

Answers

The mixed fraction 4 3/8 + 7 3/4 is equivalent to the fraction to 12 1/8

How to solve an equation?

An equation shows the relationship between two or more numbers and variables.

The mixed fraction 4 3/8 is equivalent to the fraction (4 * 8 + 3)/8 = 35/8

The mixed fraction 7 3/4 is equivalent to the fraction (4 * 7 + 3)/4 = 31/4

Hence:

4 3/8 + 7 3/4

Substituting:

= 35/8 + 31/4

Simplifying:

= 97/8 = 12 1/8

4 3/8 + 7 3/4 is equal to 12 1/8

Find out more on equation at: https://brainly.com/question/2972832

#SPJ1

Mr. Jones and Mr. Evansbee both have gardens shaped as squares. The length of each side of Mr. Jones’ garden is 6 yd. The length of each side of Mr. Evansbee’s garden is 10 yd.

(a) Write an expression with exponents for the area of Mr. Jones’ garden. Then, find the area of the garden.
(b) Write an expression with exponents for the area of Mr. Evansbee’s garden. Then, find the area of the garden. Each plant takes up 2 yd(squared) of space in both gardens.
(c) How many plants can Mr. Jones place in his garden? How many plants can Mr. Evansbee place in his garden? Explain your reasoning.
(d) For each garden, the cost per plant is $4.99. Using your answers from part (c), how much will each gentleman spend on plants for his garden?

Answers need to be step-by-step please

Answers

Answer:

  (a) expression: (6 yd)²; area: 36 yd²

  (b) expression: (10 yd)²; area: 100 yd²

  (c) Jones: 18; Evansbee: 50

  (d) Jones: $89.82; Evansbee: $249.50

Step-by-step explanation:

You want the area, number of plants that can be placed, and the cost of plantings for gardens that are 6 yards square and 10 yards square. Each plant takes 2 square yards and costs $4.99.

Area of a square

The formula for the area of a square of side length s is ...

  A = s²

(a) Jones' garden

The area of a square garden with a side length of 6 yards is ...

  A = (6 yd)² . . . . . expression for the area

  A = 36 yd² . . . . . the area of the garden

(b) Evansbee's garden

The area of a square garden with a side length of 10 yards is ...

  A = (10 yd)² . . . . . expression for the area

  A = 100 yd² . . . . . the area of the garden

(c) Number of plants

The area used by the plants is the product of their number and the area taken by each.

For Jones' garden, the number of plants that can be placed is ...

  n(2 yd²/plant) = 36 yd²

  n = (36 yd²)/(2 yd²/plant) = 18 plants

Mr. Jones can place 18 plants in his garden.

For Evansbee's garden, the number of plants that can be placed is ...

  n(2 yd²/plant) = 100 yd²

  n = (100 yd²)/(2 yd²/plant) = 50 plants

Mr. Evansbee can place 50 plants in his garden.

(d) Cost of plants

At $4.99 each, the cost of plantings will be $4.99 times the number of plants.

Plants for Jones' garden cost 18($4.99) = $89.82.

Pants for Evansbee's garden cost 50($4.99) = $249.50.

Pls help i give crowns

Answers

Answer:

1. 27

2. [tex]\frac{27}{7}[/tex]

Step-by-step explanation:

9 / 7/3

9 * 3/7

9/1 * 3/7

27/7

Answer:

The answer to the first question is:
[tex]27[/tex] groups.

And the second answer is:
[tex]\frac{27}{7}[/tex]

Step-by-step explanation:

Step 1: Analyze the number line

As we can see from the number line, each interval equals to [tex]\frac{1}{3}[/tex].

We can prove this by the fact that at the seventh interval, the value is [tex]\frac{7}{3}[/tex].

So, the value between each intervals would be:

[tex]\frac{7}{3}\div 7\\=\frac{1}{3}[/tex]

Step 2: Solving the first question:

We can see that the number [tex]9[/tex] lies on the [tex]27\text{th}[/tex] interval.

Since the value between each interval is [tex]\frac{1}{3}[/tex], it would take [tex]27[/tex] groups of [tex]\frac{1}{3}[/tex] to equal to [tex]9[/tex].

Step 3: Solving the second question:

From the number line, we can see that it takes four groups of [tex]\frac{7}{3}[/tex] to equal to [tex]\frac{28}{3}[/tex].

And since we are taking [tex]\frac{7}{3}[/tex] as a single group here, each interval in a group of [tex]\frac{7}{3}[/tex] would be [tex]\frac{1}{7}[/tex] of the group.

So, the answer of the division would be: [tex]4[/tex] (the number of groups of [tex]\frac{7}{3}[/tex]needed to reach [tex]\frac{28}{3}[/tex]) minus [tex]\frac{1}{7}[/tex] (the amount of the group that when subtracted, brings the entire answer to [tex]9[/tex]):

[tex]4-\frac{1}{7}\\\\\frac{4\times 7}{7}-\frac{1}{7}\\\\=\frac{28}{7}-\frac{1}{7}\\\\=\frac{28-1}{7}\\\\=\frac{27}{7}[/tex]

Hey bb I know that it was ur b day and u said u were gonna come out to ur parents as bi. Did u do it and if u did how did it go. Just checking in, ofc u don’t know me so u don’t even have to tell me anything if you want.

Answers

An event can be said to be a set of possible outcomes which results from an experiment.

An individual can be said to create a theory after he observes events.

A theory is said to be a set of ideas that explains the results from an experiment.

Therefore, an individual creates a theory after he observes an event.

ANSWER:

Theory

Plot (−1 3/4, 4 1/2) on the coordinate plane. Keyboard Instructions Initial graph state The horizontal axis goes from -5 to 5 with ticks spaced every 1 unit(s). The vertical axis goes from -5 to 5 with ticks spaced every 1 unit(s).

Answers

The point [tex](-1\frac{3}{4},4\frac{1}{2})[/tex] has been plotted on the coordinate plane.

Given the point [tex](-1\frac{3}{4},4\frac{1}{2})[/tex]

The coordinate plane is used to graph the points, lines and other objects.

First we have to convert the mixed fraction to the simple fraction

[tex]-1\frac{3}{4}[/tex] = -7/4

[tex]4\frac{1}{2}[/tex] = 9/2

Now we have to convert the simple fraction to decimal form

-7/4 = -1.75

9/2 = 4.5

The point is (-1.75,4.5)

Here the condition of axis is given, the horizontal axis is goes from -5 to 5, that is -5 < x < 5, and the vertical axis goes from -5 to 5 that is -5 < y < 8

Plot the points on the coordinate plane  

Hence, the point [tex](-1\frac{3}{4},4\frac{1}{2})[/tex] has been plotted on the coordinate plane.

Learn more about coordinate plane here

brainly.com/question/28471473

#SPJ1

Without using a calculator, order the
following expressions from least to greatest.
29 2
3
64

Answers

The expressions from least to greatest is (√3)² < 8² < 29².

How to compare expressions with indices?
Expressions with indices are of three types:
(1) when bases are equal and powers are different;
(2) when powers are equal but bases are different;
(3) when both powers and bases are unequal.
To solve an expression of type (1), we compare the powers for the given expressions only to determine the greater number efficiently, no heed needs to be given to the base.
To solve an expression of type (2), we compare the bases for the given expressions only to determine the greater number efficiently, no heed needs to be given to the powers.
To solve an expression of type (3), we reduce the power of an expression until it matches the other, so that for comparison only bases are to be taken into consideration without any heed to powers.

Given the expressions are 29², 3 and 64
The given expressions can be re-written as: 29², (√3)² and 8².
On comparing the expressions with the above literature, they match type (2), thus following the assertions we should compare the bases only to find the greatest number.
Thus, the correct order for expressions is: (√3)² < 8² < 29²

To learn more about this, tap on the link below:
https://brainly.com/question/27810886

#SPJ9

5. If y = 1 and x = then in terms of x, y = (A)+1 (B) (C) (D)

Answers

The question states:

Notice that the angle for which its measure is arctan(a/b), is the angle that has "a" as the oposite side, and "b" as the adjacent side , so it is the angle on the bottom right of the triangle.

Then the cosine of that angle, is defined as:

cos(angle) = adjacent/hypotenuse = b / squareroot( a^2+b^2)

[tex]\cos (\theta)=\frac{b}{\sqrt[]{a^2+b^2}}[/tex]

which agrees with answer D in the list of possible answers given.

Trish's resting heart rate is 50 beats per minute. For every
minute she exercises, her heart rate increases 5 beats per
minute. How long will it take her to reach a heart rate of
120 beats per minute?
A 5 minutes
B 14 minutes
C 34 minutes
D 70 minutes

Answers

Answer:

B

Step-by-step explanation:

(120 - 50 b/m) / 5 b / m^2 = 14 minutes    

Determine the area of the shaded triangle inside the square explain your strategy

Answers

To find the area of the shaded region, we have to find the area of the square first.

[tex]A_{\text{square}}=l^2=(10in)^2=100in^2[/tex]

Then, we find the area of each yellow triangle inside (up and down). The base of each triangle is 10 inches, and the height of each of them is 7 inches.

[tex]\begin{gathered} A_{\text{triangle}}=\frac{1}{2}bh=\frac{1}{2}\cdot10in\cdot7in=35in^2 \\ A_{\text{triangle}}=\frac{1}{2}\cdot10in\cdot7in=35in^2 \end{gathered}[/tex]

Add the area of both triangles.

[tex]A_t=35in^2+35in^2=70in^2[/tex]

At last, subtract the area of the triangles from the area of the square.

[tex]A_{\text{shaded}}=100in^2-70in^2=30in^2[/tex]Therefore, the area of the shaded region is 30 in^2.

The strategy is to subtract the area of the yellow triangles (up and down) from the area of the square.

please help im stuck on this

Answers

04) The complementary angles are the angles whose sum is 90° thus { 26°,54°,48°,29°,22°} will be the corresponding angles.05)The supplementary angles are the angles whose sum is 180° thus {54°,48°,12°,135°,91°} will be the corresponding angles.

What is an angle?

An angle is a geometry in plane geometry that is created by 2 rays or lines that have an identical terminus.

The identical endpoint of the two rays—known as the vertex—is referenced as an angle's sides.

If m∠A + m∠B = 90° then A and B will be the complimentary angle.

The complementary of 64° = 90° - 64° = 26°

The complementary angle of 36° = 90° - 36° = 54°

The complementary angle of 42°= 90° - 42° = 48°

The complementary angle of 51°= 90° - 51° = 29°

The complementary angle of 68°= 90° - 68° = 22°

If m∠A + m∠B = 180° then A and B will be the supplementary angle.

The supplementary of   = 180° - 126°  = 54°

The supplementary of   = 180° -  132° = 48°

The supplementary of   = 180° - 178°  = 12°

The supplementary of   = 180° -   45° = 135°

The supplementary of   = 180° -  89°  = 91°

Hence " { 26°,54°,48°,29°,22°} will be the corresponding complementary angle and {54°,48°,12°,135°,91°} will be the corresponding supplementary angles".

For more about the angle,

brainly.com/question/13954458

#SPJ1

14. Use Structure Consider the rational
number.
a. What are the values of a and b in a)b wher
you use division to find the decimal form?
b. What is the decimal form for?3/11

Answers

The decimal form of this rational number is 0.27, where 27 in decimal is recurring.

What are the steps to solve a rational number?
The steps to solve a rational number problem are:
(a) Let 'a/b' be a rational number. From the sentence, we deduce that denotes the denominator of a rational integer, while represents the numerator. Consequently, is 3 and is 11.
(b) The decimal form is achieved by dividing by, and we described the procedure required to do so:
(i) Ten times the numerator:
Dividend is 30, and the result is known to be zero. Remainder: N/A
ii) Dividend less the dividend multiplied by the divisor by two:
Known outcome: 0.2; residuals: 8
iii) 10 times the residue:
Known outcome: 0.2; residuals: 80
iv) Divide the residue by the divisor and multiply it by 7:
0.27 as known result; 3 as residue
v) Ten times the residue:
0.27 as known result; 30 as residue
vi) Divide the divisor by two and deduct the result from the sum:
Results known: 0.272/Residue: 8
vii) 10 times the residue:
The known outcome is 0.272/Residue: 80.
viii) Divide the divisor by 7 and deduct the result from the sum:
A known outcome is 0.2727/Residue: 3.

Therefore, we can see that is a periodic decimal number in its decimal form.
As a result, we deduce that the decimal representation of this rational number is 0.27, where 27 is a regular integer.

To learn more about this, tap on the link below:
https://brainly.com/question/19053719

#SPJ9

g(x) = 4x + 4
f(x) = x3 − 1
Find (g ◦ f)(x)

Answers

Answer:

4x³

Step-by-step explanation:

substitute x = f(x) into g(x)

(g ○ f)(x)

= g(f(x))

= g(x³ - 1)

= 4(x³ - 1) + 4

= 4x³ - 4 + 4

= 4x³

Solve the equations justify your response by showing mathematical steps

Answers

Given:-

[tex]undefined[/tex]

The triangle below will be translated 2 units up and 5 units to the right.3АA22-4-3045- 1Bс2What will be the coordinates of point A'?O (4,4)O (1,7)O (-3,-3)O (-6,0)

Answers

In general, a translation:

• in the x-direction by n units is given by: (x, y) → (x + n, y),

,

• in the y-direction by m units is given by: (x, y) → (x, y + m),

,

• the combination of both translations is: (x, y) → (x + n, y + m).

In this problem, we want to translate a triangle 2 units up and 5 units to the right, so we want to translate the points of the triangle in the following way:

[tex](x,y)\rightarrow(x+5,y+2)[/tex]

From the graph we see that the coordinates of point A are:

[tex](x_A,y_A)=(-1,2)[/tex]

Making the translation we get:

[tex](x_A+5,y_A+2)=(-1+5,2+2)=(4,4)[/tex]

Answer

(4,4)

Which set of ordered pairs represents y as a function of x? А A {(2,-1),(4,-2),(6,-3),(8,-4)} B {(0,0),(1,1),(1,0),(2, 1)} © {(3,3), (3,4),(4,3),(4,4)} {(1,-5),(1,5),(2,-10),(2, -15)}

Answers

Function:

A function is a relation in which every input x has a different output value y.

If there is more than 1 outputs for the same input then it doesn't represent a function.

Let us analyze each of the given options.

Option A:

{(2, -1), (4, -2), (6, -3), (8, -4)}

As you can see, each input x has a different output value y.

Therefore, it is a function

Option B:

{(0, 0), (1, 1), (1, 0), (2, 1)}

As you can see, the input (1) has two different outputs (1 and 0)

Therefore, it is not a function.

Option C:

{(3, 3), (3,4),(4,3),(4,4)}

Which set of values belong to the domain and range of a relation?

Answers

Domain is x values of the set and range is y values of the set.

What is domain and range?

Domain is the set of all first numbers of the ordered pairs in a relation and range is the set of second numbers of the ordered pair in relation.

The domain is the set of independent values, meaning the set of x-coordinates of the ordered pair as x is an independent variable.

The range is set of dependent values, meaning the set of y-coordinates of the ordered pair as y is an independent variable.

For example- we have a set

(1,2) (0,6) (2,3) (4,5)

Domain = {1,0,2,4}

Range = {2,0,3,5}

To know more about domain and range, visit:

https://brainly.com/question/20349172

#SPJ9

Please help with both ?s I’ll mark you brainly

Answers

Answer:

  1) CW 270° or CCW 90°

Step-by-step explanation:

In each case, you want to identify two transformations that will map the preimage to the image.

1)

We note the transformation maps ...

  A(-7, 0) ⇒ A'(0, -7) . . . . . . CCW 90° or CW 270°

  B(-4, -1) ⇒ B'(1, -4)

  C(-7, 3) ⇒ C'(-3, -7)

These can be described by ...

  (x, y) ⇒ (-y, x)

This transformation is equivalent to either of the two transformations ...

rotation CW 270° about the originrotation CCW 90° about the origin

2)

As above, rotation about the origin by 90° in one direction is the same as rotation 270° about the origin in the other direction. The attached graph shows the result either way. The two equivalent transformations are ...

rotation CW 90° about the originrotation CCW 270° about the origin

The coordinate transformation is ...

  (x, y) ⇒ (y, -x)

Other Questions
A bag contains 42 red, 45 green, 20 yellow, and 32 purple candies. You pick one candy at random. Find the probability that it is not yellowP(not yellow) Why would Twain have added the term "localized" to the title of this sketch? 2 What is monumental architecture?A colorful murals that decorated the outside ofbuildingsB groups of homes built close together, withoutstreets or sidewalksC massive gravesitesD. large structures built for specific purposes Please v Ferguson which of the following statements best represents justice on Harlans argument You are conducting an experiment and create a stock solution. You then add 1 ml of the stock solution to 99 ml of water. What is the dilution of this sample? X = times more dilute)A. 0.1xB. 10xC. 100xD. 1000x Here is a linear equation: y = 2/3x + 1 Are (6, 5) and (9, 8) solutions to the equation? Explain or show your reasoning. A person without the allele for sickle cell anemia has children with a person who is a carrier for sickle cell anemia but doesnt have the disease. What percentage of their children will have sickle cell anemia? What percentage of their children will be carriers for sickle cell anemia? Show your work with a Punnett square. Queen pls I need help ASAP A $1,000 deposit is made at a bank that pays 12% compoundedHow much will you have in your account at the end of 10monthly.years? solve for v: -28 = v +-12 The amount of energy required to rise one gram of a substance, one degree Celsius is called? [Isosceles and Equilateral Triangles] Solve for x. Please explain and help me get the correct answer. Thank you. Practice work that is not graded. When the government does not use quotas, taxes or other means to restrict what its citizens can buy from or sell to another country, it is called trade. A sequence is defined recursively using the formula f(n+1) =-0.5f(n) . If the first term of the sequence is 120, what is f(5)?157.57.515 two nurses are discussing healthcare quality. they agree that this factor contributes to increased health care quality: group of answer choices non magnet status hospitals fewer adverse events collaboration of multiple healthcare agencies increased patient confidence in the united states, the mean birth weight for boys is kg, with a standard deviation of kg. assuming that the distribution of birth weight is approximately normal, complete parts a through e. 8. Tabitha made 12 out of 23 free throw attempts in the first half of practice and 20 outof 27 attempts in the second half of practice. What is her free throw average for thewhole practice, written as a decimal? micrometers, T is measured in seconds, and D accounts for the weakening of the earthquake due to the distance from the epicenter. If an earthquake occurred for 4 seconds and D = 2, which graph would model the correct amount on the Richter scale? Please help super confused ASAP. Find f(-9) if f(x) = -2x + 5. Enter DNE if the value does not exist.f(-9)= Write an equation of the parabola in vertex form.(-2, 6)(-1,3)